¿Existe la supersimetría 4D N=3N=3{\cal N} = 3?

El libro de Steven Weinberg "The Quantum Theory of Fields", volumen 3, página 46 da el siguiente argumento contra norte = 3 supersimetría:

"Para el mundo norte = 4 supersimetría solo hay un supermultiplete ... Esto es equivalente a la teoría de la supersimetría global con norte = 3 , que tiene dos supermultipletes: 1 supermultiplete... y el otro el supermultiplete conjugado CPT... Sumando los números de partículas de cada helicidad en estos dos norte = 3 supermultiplets da el mismo contenido de partículas que para norte = 4 supersimetría global"

Sin embargo, esto no implica directamente (que yo sepa) que no haya norte = 3 QFT. Tal QFT tendría el contenido de partículas de norte = 4 super-Yang-Mills pero no tendría la misma simetría. ¿Se conoce tal QFT? Si no, ¿es posible probar que no existe? Supongo que sería posible examinar todos los lagrangianos posibles que darían contenido a esta partícula y mostrar que ninguno de ellos tiene norte = 3 (pero no norte = 4 ) supersimetría. Sin embargo, ¿es posible dar un argumento más fundamental, basándose solo en principios generales como la invariancia de Lorentz, la descomposición de conglomerados, etc., que descartaría dicho modelo?

Aunque no dice esto explícitamente, su pregunta es sobre la supersimetría de Poincaré de cuatro dimensiones. Ciertamente, en tres dimensiones, hay norte = 3 teorías
Por supuesto. Cambié el título para hacerlo más preciso.
¿Alguien ha leído el cap. 12 de ["Harmonic Superspace" ebooks.cambridge.org/ebook.jsf?bid=CBO9780511535109] , titulado "N=3 teoría super Yang-Mills"? Nunca he entendido si lo que describen es N=3 o N=4
@Yuji--- Es N=4 en el caparazón.

Respuestas (3)

Dependiendo de lo que quiera decir con "existir", la respuesta a su pregunta es .

Hay un norte = 3 álgebra de supersimetría de Poincaré, y hay realizaciones de la teoría de campos. En particular, hay un cuatridimensional norte = 3 teoría de la supergravedad. Una buena referencia moderna para los diversos sabores de las teorías de la supergravedad es la Estructura de las teorías de la supergravedad de Toine Van Proeyen .

Adicional

El argumento de Weinberg es esencialmente la siguiente observación. Tome una representación unitaria sin masa de la norte = 3 Superálgebra de Poincaré con helicidad | λ | 1 . Esta representación no es estable bajo CPT, por lo que el teorema de CPT dice que para darse cuenta de que en una teoría cuántica de campos supersimétrica, debe agregar la representación conjugada de CPT. Una vez que haces eso, sin embargo, el la representación admite de hecho una acción del norte = 4 Superálgebra de Poincaré.

La razón por la que existe la teoría de la supergravedad (y es diferente de norte = 4 supergravedad) es que el norte = 3 multiplete de gravedad, que es una helicidad sin masa | λ | = 2 representación unitaria, ya es CPT-autoconjugado.

De acuerdo, aunque creo que, estrictamente hablando, la supergravedad no es una QFT, ya que una cuantificación consistente de una teoría gravitacional presumiblemente requiere algo más que una QFT, a saber, la teoría de supercuerdas.
Sí estoy de acuerdo. Pero el argumento de Weinberg es puramente cinemático. Es una propiedad de la teoría de la representación unitaria de la norte = 3 Superálgebra de Poincaré con el requisito adicional de invariancia CPT.
También, aunque norte = 3 la supergravedad probablemente no sea renormalizable, este no es el caso para todas las teorías de supergravedad. Hay muchos indicios de que norte = 8 la supergravedad es en realidad finita. Véase, por ejemplo, relativity.livingreviews.org/Articles/lrr-2002-5
Siento que hay razones profundas por las que ningún QFT puede ser una teoría de la gravedad (excepto en el sentido holográfico), pero es un tema diferente. Todavía no sé la respuesta a mi pregunta original, es decir, ¿hay un QFT (honesto, no gravitacional) en 4D con supersimetría N = 3?
Acepto la respuesta ya que, aunque la gravedad cuántica no es una QFT, la existencia de gravedad cuántica N = 3 probablemente descarta la posibilidad de un teorema de no-go en la línea de los principios básicos (ya que la mayoría de ellos no son violados por la gravedad y el principio que se viola - la localidad - solo se viola de una manera bastante sutil).
Gracias, pero en realidad, ¡creo que la respuesta que debería aceptarse en la respuesta de Paul a continuación!
Bueno, la respuesta de Paul es complementaria, ¡pero desafortunadamente no puedo aceptar ambas! :)

La discusión en las páginas 168-173 en Weinberg vol III busca excluir rígido norte = 3 QFT supersimétricas en 4d, al menos aquellas que sean renormalizables y con descripción lagrangiana.

El primer paso es notar que, para identificar el autoconjugado CPT norte = 4 supermultiplete con el norte = 3 supermultiplet más su conjugado CPT, se debe suponer que todos los campos en ambos supermultiplet se valoran en la representación adjunta del grupo de calibre. En norte = 1 lenguaje, los constituyentes básicos en ambos supermultipletes son un calibre y tres supermultipletes quirales, todos con valores adjuntos. Los tres supermultipletes quirales deben transformarse como un triplete bajo el s tu ( 3 ) parte de tu ( 3 ) R-simetría de la norte = 3 superálgebra.

Cualquier teoría de campo lagrangiana renormalizable en 4d que tenga un norte 2 la supersimetría debe tomar la forma dada por (27.9.33) en Weinberg. Esto solo corresponde al acoplamiento genérico en la carcasa de rígido norte = 2 vector e hipermultipletes, con renormalizable norte = 2 superpotencial (27.9.29). Para norte > 2 , vector e hipermultipletes deben transformarse en la representación adjunta del grupo de indicadores. ( norte = 2 requiere solo que el hipermultiplete se transforme en una representación real del grupo de calibre, es decir, una representación "no quiral" en norte = 1 lenguaje.) Poniendo en esta suposicin, el norte > 2 El caso se deduce fácilmente utilizando el análisis de Weinberg a continuación (27.9.34). Todos los términos excepto los de las dos últimas líneas de (27.9.33) se ensamblan precisamente en el norte = 4 Yang supersimétrico: lagrangiano de Mills. Los términos restantes en las dos últimas líneas de (27.9.33) dependen de una matriz m que define el término cuadrático en el superpotencial. Como argumenta Weinberg, norte = 4 ocurre sólo si todos estos términos desaparecen de forma idéntica (por ejemplo, si m = 0 ). De dónde norte = 3 puede ocurrir solo si los términos en las dos últimas líneas de (27.9.33) no se anulan y norte = 3 supersimétricos por sí solos. Esto requeriría que sean invariantes bajo el tu ( 3 ) R-simetría de la norte = 3 superálgebra. Sin embargo, solo dos de los tres supercampos quirales (procedentes del hipermultiplete) aparecen en el m -términos dependientes. Dado que los tres supermultipletes quirales deben transformarse como un s tu ( 3 ) triplete bajo la simetría R, es claramente imposible que las dos últimas líneas en (27.9.33) sean tu ( 3 ) -invariante a menos que desaparezcan idénticamente. De dónde, norte > 2 implica norte = 4 en este contexto.

El argumento de Weinberg se basa en la existencia de un régimen de acoplamiento débil/lagrangiano. Auténtico norte = 3 Las teorías se encontraron recientemente en este artículo . Como era de esperar, están fuertemente acoplados y se cree que no son lagrangianos.